« first day (2351 days earlier)      last day (2656 days later) » 

12:00 AM
@TedShifrin no, i really mean $\sum_{k=1}^{2n+1}a_n$ and NOT only all odd terms
 
OK. So I'm comparing that to $x\cdot h/\|x\|$.
 
@Null? When you put $2n+1$ in the upper limit, that's an odd partial sum.
 
mmh
 
ah, darn it
okay, i'm stuck on a step.
 
what step?
 
12:02 AM
so i've got the midpoint and the vertex
 
yeah well, then 1-1+1 obviously converges to one?
 
and all of it graphed and whatnot
 
@TedShifrin Recitations or sectons is fine. I try to teach in these, so I don't like the term problem sessions. If a student wants to talk to me about problems, I am always available.
 
Say what, @Null?
 
so i was like, okay, i'll...wait. i think i just figured it out, one moment.
 
12:02 AM
Gotcha, @MikeM. I humbly apologize :)
 
You fine.
 
okay, got it.
 
@TedShifrin first: I calm down :d . I just think you misunderstood me, which could easily be false.
 
(3,5)
fabulous.
 
If you want to double-check, you can do it for another median, @heather.
 
12:03 AM
@TedShifrin okay.
 
Then I'll tell you the secret.
 
sounds good
 
So, @Null, if we do $1-1+1-1+1-1+1-\dots$, the even partial sums are $0$ and the odd partial sums are all $1$. Is this not an example? The series does not converge.
 
$$\frac{\|x+h\|-\|x\|-h\cdot x/\|x\|} {\|h\|}=\\x\cdot\frac h{\|h\| }\left(\frac2{\|x+h\|+\|x\|}-\frac1{\|x\|}\right)$$
 
@TedShifrin $\sum_{k=1}^{2n+1}a_k$ has to converge to meet my initial "if"
 
12:05 AM
It does, @Null.
It's always $1$.
You mean $a_k$ in the sum, btw.
 
yep
ah ok
then my misconception was there
 
Which clearly goes to $0$.
 
my god, what do you have akiva doing?
@Null 1, 1, 1, 1, 1, ... are all close to 1.
 
@TedShifrin See the above?
 
Yeah, DogAteMy. I don't get your steps yet.
 
12:07 AM
okay, got it
 
@MikeMiller Showing from the definition that $\|x\|$ is differentiable, which is the same as showing that the first line goes to $0$
 
Away from zero?
 
Right. $x\ne 0$.
 
I thought, convergence of a sum means, all terms are close to zero at the end, as a neccessary, but not sufficent criterion
 
I'm off to bed now, good night or day to everyone
 
12:08 AM
@TedShifrin Just factoring out $x\cdot h/\|h\|$
 
Night, @Alessandro.
 
Using what I got earlier
 
@JessyCat what worked?
 
@GFauxPas happy birthday
 
I can't keep it all in my memory bank, DogAteMy.
 
12:10 AM
@TedShifrin, ready for the secret =D
 
It's at least off by a minus sign, DogAteMy, isn't it?
 
@AlessandroCodenotti that is correct.
 
@heather: Find for me the average of the three vertices.
 
We know $\|x+h\|-\|x\|=2x\cdot h/(\|x+h\|+\|x\|)$. Therefore, $$\frac{\|x+h\|-\|x\|-h\cdot x/\|x\|} {\|h\|}=\\x\cdot\frac h{\|h\| }\left(\frac2{\|x+h\|+\|x\|}-\frac1{\|x\|}\right)$$
 
okay
 
12:10 AM
@SteamyRoot I saw your answer. Thanks. I'll have to take a closer look at it. Uses the centralizer version.
 
@TedShifrin so, to sum up the gist, convergence of the even/odd terms, says not much about the others?
 
Right, except in the case you proved 2 hours ago, @Null :)
 
@robjohn your directions on how to run chatjax on my iPad
 
@JessyCat Ah. Yes. I use it on my iPhone, too.
 
@robjohn: You kindly put the directions that I dug up from some clever source.
 
12:12 AM
@TedShifrin okay, i believe i did it correctly...just $\frac{x_1+x_2+x_3}{2}$ and the same for the y's, right?
 
Yup, @heather.
 
okay.
 
@robjohn I'm going to do the same on there.
 
OK, DogAteMy. I'm convinced. Well done. (Of course, the first coefficient is bounded, and the second term goes to 0.)
 
12:14 AM
Pretty cool, eh, DogAteMy?
I thought you might like that because you're good at tricky high school algebra.
I'm sure you have important stuff to do, so I won't give you more.
 
@TedShifrin but I just want to be sure, $\sum_{k=1}^{2n+1} (-1)^k$ is convergent, if the top value is even? But isn't convergence defined this way: for any epsilon, we can choose an integer, such that after that, the sum is smaller then L+$\epsilon$?
 
So we were only working on normed spaces?
Inner product spaces would be about twenty times easier. :)
 
@MikeM: No, we had a dot product there.
 
Oh, what? Why didn't we use it?
 
We did.
 
12:17 AM
It seems like you did way too much work.
 
@Null: The point is that for the entire series to converge, the even partial sums and the odd partial sums must have the same limit.
DogAteMy did the work, not I.
He expanded the difference of squares using dot product.
 
sweet, did another problem
i'm getting the hang of this =D
 
@heather: Wait! What did you get for your average?
 
$\|x+h\|^2 - \|x\|^2 = 2h \cdot x + h \cdot h$. Dividing out by magnitude of $h$, we get $2h \cdot x / \|h\|$, which is well-defined in the limit. Thus one concludes that $\|x\|^2$ is differentiable, and so taking square roots we're good away from zero.
Apparently that's what he did but his work looked way more complicated.
 
@TedShifrin, the original points were (-1,11),(7,6), and (3,1) and I got for the average (4.5,9)
 
12:20 AM
Huh? We don't have the chain rule yet, @MikeM.
@heather: Then something's wrong.
First principles, @MikeM.
 
Oh wow I don't care that much.
 
@MikeMiller The problem is that I haven't shown that $\|x\|^2$'s differentiability gives us $\|x\|$'s.
 
No 'ffense. I see why it's worse.
 
Well, it wasn't meant for you, @MikeM.
 
<3~
 
12:21 AM
@heather: recompute your average, please.
 
@TedShifrin, okay
 
Xam
Hey folks, what are you talking about?
 
oh, duh, i know what i did wrong
 
I do too.
 
(3, 8/3)
 
12:22 AM
GRR.
 
is what i got this time.
no?
 
18/3 = 6
 
ack, what's wrong with me...
wait, wait, let me do that again
i can't add
 
Hmm, we're supposed to get the centroid. So I wonder what you did before.
 
Xam
mmm
 
12:23 AM
Well, off to sleep. Night all.
 
(3,6)
 
Night, @Steamy.
 
there we go.
 
Is that what your centroid was?
 
which is different from the centroid...
no, i got (3,5)
 
12:24 AM
Well, it shouldn't be. You ruined my surprise!! :D
 
but my mistakes up to this point clearly point the finger at my centroid result
::shakes head in shame::
 
LOL
 
this is not a good mathing day for me.
we spent all of 5 minutes on finding the centroid...
ergh.
 
So double-check your arithmetic. The centroid is in fact the balance point of the triangle.
That's the arithmetic mean of the vertices.
 
yeah. that does make sense now that i think about it.
 
12:25 AM
It's also true that if you made a triangle out of metal it would balance at the same point.
I get (3,6) when I do the point 2/3 down a median.
 
@TedShifrin but which integer do you choose, such that all terms after it lie in the epsilonrange?
 
this is so bad, i got a different answer that wasn't 3,6
::face palms::
lemme do this again
 
Start at (-1,11) and go 2/3 the way to (5,7/2).
@Null: For our example, you cannot. The series does not converge. That's the point.
 
oh, i was doing the midpoint of AC, so 3, 17/2 to 3,1
 
The even partial sums are 0 and the odd partial sums are 1. Those aren't very close.
 
12:29 AM
or just do 2/3s the distance between 17/2 and 1, or, 2/3*15/2, where i got 3, which is wrong...
i'm doing something really wrong here.
 
The midpoint of AC is (1,6).
And then you go from B to that.
 
what the heck is wrong with me
okay, lemme try finding the midpoint again
how are you doing that? I don't understand
-1+7 = 6, divided by 2 is 3
that's the x coordinate
 
Oh, that's AB, not AC.
(You typed the three points above.)
 
@TedShifrin but my initial statement was "if $\sum_{k=1}^{2n+1}a_k$ converges for $n\to\infty$, then $\sum_{k=1}^{\infty}a_k$ converges too. "
 
::facepalms again::
 
12:31 AM
That's false, @Null. We have the counterexample.
 
okay, the midpoint of AB is (3, 17/2)
 
You proved that if the odd partial sums converge AND $a_k\to 0$, then we're fine.
 
i did that right
 
LOL. Go on.
And now go 2/3 the way from C to that point.
 
okay, so since the x coordinates are the same, you just need to subtract the y coordinates.
so 17/2-1
 
12:33 AM
Right.
 
or 15/2
 
@TedShifrin my point is, that 1-1... doesn't converge, so it doesn't fullfill my "if". :/
 
okay, so then 2/3 of that, so 2/3*15/2
or 3
 
But you have to add that to the original y-coordinate at C.
@Null: You are just getting distracted and keep making the same mistakes.
 
12:34 AM
wait, i thought add that to 17/2
because shorter part is between the centroid and midpoint.
 
No, you're going FROM C to the midpoint.
We went 2/3 the way from the vertex to the midpoint.
 
doing that means you do 1+3=4. or you get (3,4) for the centroid. that's wrong.
 
LOL ... 5+1 = 6.
 
wait, what
where'd you pull the 5 from?
 
12:35 AM
We start at 1 and add 2/3 * 15/2 = 5.
 
my point above: 2/3*5/2 =/= 3
 
@TedShifrin I keep making it, because I don't understand what's wrong. Doesn't convergent mean, that all terms are close to zero. (for a sum)
 
OH
wow.
this is not a good day.
 
You're confusing convergence of a sequence and convergence of a series, @Null, I think.
 
lemme do the multiplication again.
 
12:36 AM
Cancel before you multiply? :)
 
got it.
3,6.
 
Yippee ;)
 
there we go.
same as the averages!!
 
Practice a bit more. Anyhow, you sorta ruined my punchline, but oh well :D
 
@TedShifrin, it still was magical =P thanks for the help though
 
12:36 AM
That's why the centroid is important — it's the balance point.
 
@TedShifrin if $\sum a_k$ converges, $a_n\to 0$ for $n\to\infty$, or not?
 
i will try to avoid more addition errors.
(and multiplication...and subtraction...and division)
 
Yes, @Null, that is true.
 
If $a_n$ doesn't converge to zero, then there's no way the series can converge.
 
But that's assuming the sequence of all partial sums converges.
 
12:37 AM
so, why ever find the centroid the longer way when you can do it the other way?
 
Right. But we do not have a convergent series. We have only the odd partial sums that converge (and even, too, in our case).
@heather: You'll recall I asked you for your definition of the centroid. There's a theorem going on here.
 
@TedShifrin, um, to be honest, i'm not sure my teacher cares how i do it, though.
 
But you want to be a good mathematician, @heather, so ... :D
If you are adding better tomorrow, you can actually prove the theorem.
 
@TedShifrin, ack, when people make good points like that =P
ooh, proofs are fun
 
Just put letters in for the vertices and do the arithmetic correctly and you'll have a proof of the theorem!
 
12:40 AM
Sign of being trained as a physicist: I know moments of inertia better than I know centroids.
 
Rotation is more interesting than translation, @Semiclassic? :D
 
Pretty much.
 
@TedShifrin, really? cool! I'll try it when I'm done with my math and language arts homework.
 
@TedShifrin so now.. does it even make sense to talk about convergence if we only observe the odd "steps" (NOT terms, steps i mean)
 
Not remotely, @Null.
By steps you mean the odd partial sums.
Keep me posted, @heather :)
 
12:42 AM
will do @TedShifrin
 
Typically the objects in physics have obvious centroids, so it's the moment of inertia that comes into play in mechanics problems.
 
i just nailed two more problems (they had some nice, convenient zeroes.)
 
Sure, @Semiclassic. As my refrain in multivariable goes, "Exploit symmetry!"
 
Quite.
 
Do you teach your kidlets the parallel axis theorem?
 
12:43 AM
Not really, but I'm not in a position to do that as a TA.
 
I have that as an exercise in my book.
 
Parallel axis theorem is good stuff, yeah.
Ok, back later.
 
but if I have proven, that the limit of an odd(lets call this integer c) partial sum is something. And $s_{c-2}<s_{c-1}<s_c$ can be shown, have we proven that ??? lost my red thread haha
 
I think that will converge @Null. The convergence of odd partial sums $S_{2k + 1}$ together with the partial sums increasing would imply partial sums bounded above I believe
So monotone convergence comes in, so the partial sums converge.
In general, if $S_{2k+1}$ converges and $S_k$ also converges, then the limits will be the same
But in general, convergence of odd-indexed terms doesn't imply convergence of the whole thing.
 
What you just said is false in general, @Kaj.
We've spent 4 hours on this question.
 
12:52 AM
:S
 
@TedShifrin and I love you for spending so long on this haha
 
Oh, I haven't been here
 
If you have all the terms positive, then you're right.
 
Oh yeah, I was implicitly assuming that
Oops
 
Monotonicity of the partial sums is all terms positive :)
I don't know what you mean in your sentence, though, @Null. What integer is $c$?
 
12:53 AM
so... to not use AM-GM, I would have to use an argument like that?
an odd one
or even, really respectivly
 
I don't know anything about your original question (and I don't want to). :P
 
Man...19,999
 
Huh?
Is that your rep?
 
allllllllmost there
Yeah
 
@KajHansen should i downvote you for lols?
that you have exactly 20k
 
12:55 AM
math homework: done =D just english, and then a proof awaits...
 
If you do, do it 4 times @Null. I want to be back at $\equiv 0 \pmod{5}$
Ugh, it wouldn't be 4 times
 
@KajHansen meh, i think this triggers the system haha
 
smacks @Kaj
 
It'd be 5 times.
 
Give me a good question/answer to upvote.
 
12:57 AM
haha, that feels dirty @TedShifrin. I was excited about 20k, but it turns out it's 25k I am excited about. I really want access to the Google analytics
 
Want to borrow my rep? :)
 
is there a nice appraisal for $\frac{1}{\sqrt{n}}<??$
 
:D
What do you mean by appraisal?
Like "how large does $n$ need to be to push that less than epsilon?"
 
estimate
 
My question precisely. OK, I upvoted, @Kaj. I suppose that's a good question :D
 
12:59 AM
Ahhh
 
Write it down, @Null, and do the algebra?
 
I managed to get 40pts rep just now-ish
 
DogAteMy: I figured you had "real" homework to do.
Are you doing any math at school, or should we try to get you credit for my course? :D
 
@Null, this is a HUGE question in computer graphics
You would be interested in reading this wiki article: en.wikipedia.org/wiki/Fast_inverse_square_root
 
@Kaj: You mean Yuge?
 
1:00 AM
LOL
 
mmh, no, i try to prove $\sum\frac{(-1)^n}{\sqrt{n}}$, but meh
it's no biggy
 
fast inverse square root is used in some lighting models
 
Oh, @Null ... the point is that we don't know the limit, so we can't do the typical epsilon thing.
 
could use alternating test, yes!
 
if I recall quake implements some crazy thing
 
1:00 AM
You can most often prove a series converges without knowing to what it converges.
 
In particular, one can compute $1/\sqrt{n}$ rather precisely using only bit-shifting and addition/subtraction operations
Honestly stunning
 
He meant the alternating series, @Kaj.
 
Awwww
 
Don't worry @kaj I get you
 
1:01 AM
BTW, @Kaj, sorry we didn't get to get together this visit. If you don't come to CA, let's try my next visit.
 
That's no problem @Ted. I do wish to move out west in the nearish future
 
@KajHansen what you mean with bit-shifting?
 
I'm growing weary of the South :/
 
Well, you're welcome to visit.
I bet.
 
@TedShifrin I've been going through Rudin in school.
 
1:02 AM
Oh, really? DogAteMy ... Cool. Should I send you an exam to do? :D
 
I actually have an exam on it in my bag.
 
Is your teacher as mean as I am? :D
 
Haha. :D That's a bit of a loaded question.
 
You're going to be through most of a college math degree before you graduate high school.
 
1:03 AM
Is it? :D
Oh, you mean I need to be meaner, DogAteMy? I can do that!
 
@TedShifrin at the end I am in my exam, I see those excercises as a filter, to save paper and work for the correctors.
@KajHansen mmh, is cout >> a bitshift too? :O
 
@TedShifrin Not that you need to be meaner, just that I don't think you're as mean as you think you are :D
 
Well, that's because you like hard math, DogAteMy. Just ask Kaj. He thought I was plenty mean :)
But I'll make sure you do the hardest problems, for sure :P
 
@Null nope thats an operator overload because screw anybody reading it
 
Whenever I see a UMD address, I always think of the wrong University.
 
1:07 AM
I'm not super familiar with C languages @Null. I think cout is just like a print command
Could be wrong
 
its like a concatenation almost
 
@Semiclassical What else would it be?
 
in c++ you have cout and you << with a string
 
The explanation is rather provincial.
 
none of that nonsense in c
 
1:07 AM
@KajHansen yeah, its print something on the screen. but it uses those arrows. getchar too afaik
 
Technically, I'm at the Twin Cities campus of the University of Minnesota (UMN).
There are other campuses across the state, though the TC one is the main one.
The second biggest one? University of Minnesota Duluth.
 
So I reflexively tend to associate UMD with that. Again, very much a local thing.
 
Riight
What math are you up to lately?
I've been on a research question which starts simple and gets complicated fast.
 
1:18 AM
ok: alternating series test for $\sum(-1)^k\frac{1}{\sqrt{k}}=\sum(-1)^ka_k$. The limit of $a_n$ is 0, check. Then we have to check if |$a_{n+1}|\leq |a_n|$. Simple stuff gives us that. So it converges by the alternating series test. done?
 
Sounds good.
 
In fact, it converges absolutely since $\sum_k k^{-1/2}$ converges
 
can some one help me with a derivative im confused with - im trying to find the time when velocity of my object is zero. but from my understanding there are 2 times when that is the case. that or i am misunderstanding how to work it out. i got my derivative to be v = 3t^2 - 6t - 9 , v = 0 when t is -1 or 3 seconds right? but the question is worded in a way that i only need to find a single moment in time which is confusing me
 
Depends on context.
If it wouldn't make sense to have a time less than zero, don't do t=-1.
 
1:23 AM
well 1 seconds before is usually not wanted
 
s = t3 −3t2 −9t was the original displacement provided
true but it doesnt specify any rule of range
 
Still not enough context.
 
like there is no >= 0 after it
 
Well if it says when is the velocity 0 does saying a second ago make much sense
when its 0 in 3 seconds
which one is more important do you think?
 
fair point
but what if it was a hypothetical time wizard for a computer game :P
 
1:24 AM
As I say, it depends on the context of the problem. If they say s(t)=blah and to find when the object is at rest, then both are relevant.
 
typically you don't consider what happens with t < 0
the lack of context kind of implies this is the case
 
I'd include the t<0 one if it doesn't indicate that it doesn't matter.
 
whats a good way to write it to say -1 is not a logical choice
 
@Semiclassical i don't think so, as $\frac{1}{\sqrt{n}}>\frac{1}{n}$, and the latter diverges
 
what is the exact wording of the question
 
1:27 AM
@Semiclassical Real analysis and multivariable analysis
 
"Find the time when the velocity of the object is zero"
 
@Semiclassical What question?
 
You're right, I'm being silly. I was mislead by the fact that Mathematica gives a finite value for Zeta[1/2].
(but it's also a negative value, so it's evidently not $\sum_k k^{-1/2}$)
 
@WDUK in that case the positive time is the one they want
 
okay
 
1:28 AM
I find that a poorly constructed question.
 
okay. proof time @TedShifrin =D
finally done with language arts.
 
The classic example where you end up with a situation with two solutions is someone running at a constant speed trying to catch up with an accelerating bus.
If that speed is high enough, then you'll run right past the bus; eventually, the bus will pass again.
But if all you want is the time to catch the bus, then that second solution is irrelevant.
ok, time to go shovel snow :/
 
1:42 AM
prnt.sc/dtmw4p what is the cauchy product of this with itself? using the definition here: proofwiki.org/wiki/Definition:Cauchy_Product can I even apply the lower stuff? what the heck :/
 
user228700
Hello, everyone :-)
 
Go for it, @heather!
 
user228700
I have a very quick question about the definition of periodic functions. My textbook says that the period of any function, $T$ must always be greater than zero while other sources that I have found say that it is sufficient that $T \ne 0$. The latter seems to be correct but I wanted to make sure; so which is it?
 
@Kaumudi.H, hello
@TedShifrin =)
 
user228700
@Heather: Oh, hi! Didn't expect to see you here! :-)
 
1:53 AM
@Kaumudi.H, asked a question about centroids and discovered i cannot math today =P
 
user228700
:-P I see...
 
@heather: Name the points with coordinates and calculate the centroid. You'll get it :)
 
@TedShifrin, I'm about halfway through.
 
Oh cool :)
Just don't be silly :)
 
lol, yep
 

« first day (2351 days earlier)      last day (2656 days later) »